¿Por qué no hay un ángulo theta (término topológico) para las interacciones débiles?

¿Por qué no hay un análogo para Θ QCD para la interacción débil? ¿Se genera este término topológico? ¿Si no, porque no? ¿Está esto relacionado con el hecho de que S tu ( 2 ) L ¿está roto?

Buena pregunta, y espero con ansias las respuestas, si las hay. ;-)
Hay un ángulo de vacío electrodébil θ mi W , pero como se menciona a continuación, se puede girar en el modelo estándar gracias a la simetría quiral en el sector electrodébil. Para una buena discusión reciente, consulte, por ejemplo , arxiv.org/abs/1402.6340 . Sin embargo, cuando vamos más allá del SM, este no es necesariamente el caso. Shifman y Vainshtein demostraron recientemente que, por ejemplo, en GUT θ mi W es físico y tiene el mismo valor que θ q C D : arxiv.org/abs/1701.00467
Como comentario final, la referencia estándar con respecto a esta pregunta es ¿Puede ser observable el término θ electrodébil? por AA Anselm y AA Johansen sciencedirect.com/science/article/pii/0550321394903921
física relacionada.stackexchange.com/q/91535/12813

Respuestas (1)

En presencia de fermiones quirales sin masa, un θ El término en se puede rotar mediante una transformación quiral apropiada de los campos de fermiones, porque debido a la anomalía quiral, esta transformación induce una contribución a la medida integral del camino del fermión proporcional a la θ término lagrangiano.

ψ L mi i α ψ L

D ψ L D ψ L ¯ D ψ L D ψ L ¯ Exp ( i α gramo norte F 64 π 2 F F )

Entonces la transformación cambia θ por C α gramo norte F ( gramo es la constante de acoplamiento, norte F el número de sabores).

Los gluones tienen el mismo acoplamiento con los quarks diestros y zurdos, y una rotación quiral no deja invariable la matriz de masa. Por lo tanto, la QCD θ el término no se puede rotar.

los S tu ( 2 ) L Sin embargo, los campos están acoplados solo a los componentes izquierdos de los fermiones, por lo que tanto los componentes izquierdos como los derechos pueden girar con el mismo ángulo, girando alejándolos. θ plazo sin alterar la matriz de masa.

Genial, ¿añadirías una o dos fórmulas? ¿Cuál es el parámetro de la transformación (y cuál) para eliminar el θ F F ¿término? Y una pregunta relacionada: ¿hay alguna manera simple de agregar algunos acoplamientos quirales de nuevos fermiones a S tu ( 3 ) C o yo o r para resolver el fuerte problema CP?
@Luboš No soy un experto, solo por leer, creo que su sugerencia está bastante cerca de una solución al fuerte problema de CP, suponiendo que la masa del u-quark es exactamente cero, aunque no ampliamente aceptada.
¿Qué pasa con los acoplamientos Yukawa? ¿Absorbes la fase en el Higgs? ¿O en fermiones diestros?
@Thomas: A los fermiones diestros. No están acoplados a los campos de calibre por lo que su transformación no cambia la medida integral de trayectoria.
Hay algunas cosas básicas que no entiendo. ¿Por qué los fermiones LH y RH no pueden rotar de la misma manera en QCD? ¿Y por qué todos los sabores tienen que rotar al mismo tiempo?
@karlzr Las interacciones débiles solo se acoplan a los fermiones LH y, por lo tanto, una transformación de la RH no tiene efecto en la medida, es decir, no cambia θ w mi a k . Para QCD, tanto el trafo de la RH como el de la LH conducen a cambios separados de θ q C D , porque los gluones se acoplan a los quarks RH y LH. Lo crucial es que tenemos una libertad extra en el sector débil para rotar los fermiones RH, porque a los bosones débiles no les importan, y esto nos permite establecer θ w mi a k 0 . En QCD no tienes esta libertad.
@karlzr Para resumir: tenemos dos lugares donde las rotaciones marcan la diferencia: para los términos de masa y para los θ término. En ausencia de fermiones sin masa, no podemos realizar rotaciones quirales arbitrarias, porque los términos de masa deben ser reales. Esto significa inmediatamente que no podemos rotar θ q C D a cero. (Por supuesto, podría ser milagrosamente que la rotación que hace que los términos de masa sean reales, cancela al mismo tiempo θ q C D . Sin embargo, no hay ninguna razón por la que este sea el caso y este es el rompecabezas de CP fuerte).
@karlzr En el sector débil la situación es casi la misma. Sin embargo, los bosones débiles no se preocupan por los fermiones RH y, por lo tanto, no hay cambio de θ w mi a k cuando los rotamos. Esta libertad adicional nos permite hacer que los términos de masa sean reales (o dejarlos reales) mientras rotamos al mismo tiempo θ w mi a k a cero.
¿Entonces no recuperarías un θ ángulo en las teorías GUT?
@MichaelAngelo Para ser específico, consideraré el modelo SU (5) Georgi-Glashow. Aquí los fermiones pertenecen a la 5 ¯ a 10 representaciones. La anomalía neta de estas representaciones se desvanece, por lo que no hay forma de rotar todo el término theta. Después de romper la simetría, el término theta se descompondrá en términos theta del W -bosones, QCD, bosones de gran calibre y posiblemente combinaciones de ellos. los W El término theta de los bosones se puede rotar como se indicó anteriormente mediante una transformación quiral, el término theta QCD permanecerá así como los otros términos en la descomposición. ...
@MichaelAngelo No conozco un trabajo detallado que analice las implicaciones de estos términos, pero posiblemente puedan afectar los espectros de monopolo y dyon de la teoría a través del efecto Witten. Aquí hay dos artículos que analizan estos efectos: arxiv.org/abs/1711.05721 cds.cern.ch/record/455906/files/0008322.pdf
@JakobH Solo puede rotar el sector izquierdo y derecho en el mismo ángulo (que es necesario para que la fase desaparezca en el término de masa) si tiene un tu ( 1 ) simetría. los tu ( 1 ) Y hace esto por ti, pero en QCD no tienes un tu ( 1 ) simetría por lo que siempre tiene que girar el sector L y R con ángulo opuesto. Esto es esencial, ¿verdad? ¿Es esto correcto?
Para la parte electrodébil, en la teoría ininterrumpida, creo que el campo de Higgs no se acopla al neutrino dextrógiro (si existe), y se acopla al neutrino dextrógiro en un S tu ( 2 ) L manera invariante, y por lo tanto es posible hacer una rotación quiral sobre ellos para cancelar la θ término; en la teoría rota, la rotación quiral del neutrino sin masa hace el trabajo. Mientras que en el QCD, el campo de Higgs es un singlete bajo S tu ( 3 ) , lo que hace que su acoplamiento a los quarks no sea quiralmente invariante.